Difference between revisions of "2003 AMC 12A Problems/Problem 23"

(Solution)
Line 1: Line 1:
== Problem ==
 
 
If <math>a\geq b > 1,</math> what is the largest possible value of <math>\log_{a}(a/b) + \log_{b}(b/a)?</math>
 
 
<math>
 
\mathrm{(A)}\ -2      \qquad
 
\mathrm{(B)}\ 0    \qquad
 
\mathrm{(C)}\ 2      \qquad
 
\mathrm{(D)}\ 3      \qquad
 
\mathrm{(E)}\ 4
 
</math>
 
 
== Solution ==
 
 
Using logarithmic rules, we see that
 
 
<cmath>\log_{a}a-\log_{a}b+\log_{b}b-\log_{b}a = 2-(\log_{a}b+\log_{b}a)</cmath>
 
<cmath>=2-(\log_{a}b+\frac {1}{\log_{a}b})</cmath>
 
 
Since <math>a</math> and <math>b</math> are both positive, using [[AM-GM]] gives that the term in parentheses must be at least <math>2</math>, so the largest possible values is <math>2-2=\boxed{0}.</math>
 
  
 
== See Also ==
 
== See Also ==

Revision as of 13:55, 28 February 2010

See Also

2003 AMC 12A (ProblemsAnswer KeyResources)
Preceded by
Problem 22
Followed by
Problem 24
1 2 3 4 5 6 7 8 9 10 11 12 13 14 15 16 17 18 19 20 21 22 23 24 25
All AMC 12 Problems and Solutions